Find the slope of the line.

4/3

-4/3

-3/4

3/4

 Find The Slope Of The Line.4/3-4/3-3/43/4

Answers

Answer 1

Answer:

-3/4

Step-by-step explanation:

slope is rise over run so you go down 3 first is is -3, and then right 4 times which  means the slope is -3/4

Answer 2
Answer is -3/4 because you gotta do rise over run so you do -3 over 4 and that’s your slope

Related Questions

what is (8*8*8) * (8*8*8*8) in exponential form?​

Answers

Answer:  8^7

The exponent 7 tells us how many copies of "8" are being multiplied together.

The expression 8*8*8 is equal to 8^3, while 8*8*8*8 = 8^4

Multiplying 8^3 and 8^4 will have us add the exponents to get 8^7. The base stays at 8 the entire time.

The rule is a^b*a^c = a^(b+c) where the base is 'a' the entire time.

Answer:

8^ 7

Step-by-step explanation:

(8*8*8) * (8*8*8*8)

There are 3 8's times 4 8's

8^3 * 8^4

We know that a^b * a^c = a^ (b+c)

8 ^ ( 3+4)

8^ 7

Given f(x) = -4(x-3)^2-2
a) Determine the inverse of f(x)
b) List the domain and range of inverse.

Answers

Answer:

[tex]\sqrt{\frac{x+2}{-4} }+3=y[/tex] d=(-infinity,-2) r=(3,+infinity)

Step-by-step explanation:

i did the math

It’s d=(-infinity,-2) and r=(3, plus infinity) :) hope this helped

Jill paid $36 for calendars marked down $5 apiece from the original price, she could have gotten 5 fewer calendars. How many cards did she buy

Answers

Answer:

she bought 7 cards

Step-by-step explanation

7x5=35 and im sure there is tax

She bought 7 cards .. I say that bc if you divide 36 and 5 you’d get 7 and 5x7 is 35

The box plots display the same data set for the number of touchdowns a quarterback threw in 10 seasons of play. Including outlier: A number line goes from 5 to 30. The whiskers range from 5 to 29, and the box ranges from 18 to 26. A line divides the box at 21.5. Excluding outlier: A number line goes from 5 to 30. The whiskers range from 17 to 29, and the box ranges from 19 to 27. A line divides the box at 21. There is an asterisk at 5. Complete the statements describing the effect of the outlier on the measures of variability. The outlier of the data set is . The range of the data set including the outlier is more than the one excluding the outlier. The interquartile range of the data set including the outlier is more than the one excluding the outlier. The outlier had the most effect on the .

Answers

Answer:

5

12

0

range

Step-by-step explanation:

i just did it, these are the right answers.

Answer:

5,12,0, and the last answer is range

Step-by-step explanation:

Did it on Edge2021. Hope this helps!

Two functions, A and B, are described as follows: Function A y = 9x + 4 Function B The rate of change is 3 and the y-intercept is 4 How much more is the rate of change of function A than the slope of function B? 3 6 2 9

Answers

Answer:

6

Step-by-step explanation:

Function A

y = 9x + 4

Function B

y = max+b

m = 3 and b = 4

y = 3x+4

The difference in the slopes is 9-3 = 6

Answer:

6

Step-by-step explanation:

The two functions are A and B.

A's equation is khown

● y =9x + 4

B is also khown. We should only gather tge information.

● the rate of change is 3

●the y-intercept is 4

So B's equation is:

● y = 3x + 4

3 is the rate of change wich is khown as the slope.

Divide A's slope by B's slope to khow how much A's slope is bigger than B's.

● 9/3 = 3

Substract 3 from 9 and you get the difference 9-3= 6

The two points on the coordinate plane represent Jane's house and her friend's house. Find the distance between the houses. Question 3 options: A) units B) units C) units D) units

Answers

Answer:

8.3

Step-by-step explanation:

To do this use the distance formula

[tex]d = \sqrt{(x2-x1)^2+(y2-y1)^2}[/tex]

First find the coordinates Jane (-5, -2) Friend (3, -4)

Now solve for d (8.246)

Answer:

10 units or approx. 8.25 units

Step-by-step explanation:

Jane's house is at coordinate (-5,-2) and her friend's house is at coordinate (3,-4).

You can find the answer in 2 ways.

1) Hypotenuse (the diagonal side or side across from the right angle) - approx. 8.25 units

2) Down and Side (go down 2 units and go side 8 units) - 10 units

Hope that helps and maybe earns a brainliest!

Have great day ahead! :)

If we transform the parabola y=(x+1)^2+2 by shifting 7 units to the right and 5 units down, what is the vertex of the resulting parabola? Vertex of resulting parabola: (__ a0,__ a1)

Answers

Hey there! I'm happy to help!

The vertex form of a parabola is y=a(x-h)²+k. The h represents the horizontal transformation, while the k represents the vertical. The vertex of a parabola in this form is (h,k). The a represents a vertical stretch or shrink.

Our parabola is y=(x+1)²+2. This is already in vertex form, so we do not need to change the equation. There is no a (basically a=1), which means that the parabola has not been stretched or shrunk from its parent form.

We see that the h is -1. This is because in the original equation it is (x-h)², so it has to be -1 because the two negatives make a positive which is the +1. (x--1)=(x+1).

We see that the k value is 2.

Since the vertex is (h,k), this vertex is (-1,2).

However, we have to shift the parabola 7 units to the right and 5 units down. So, we add 7 to the x-value and subtract 5 from the y-value of the vertex.

(x,y)⇒(x+7,y-5)

(-1,2)⇒(6,-3)

Therefore, the vertex of the resulting parabola is (6,-3).

Have a wonderful day! :D

I did the math and the other comment is correct

The coefficient of 6x is
1
6
Х

Answers

Answer:

6

Step-by-step explanation:

If a number and a variable were together in a term, the number would the the coefficient. The coefficient would multiply the variable.

In '6x', the number '6' is the coefficient. '6' would be multiplying 'x'.

The correct answer should be 6.  

The coefficient of 6x is 6. This is because the coefficient is before the variable and 6 is before x.

35= -5(2k+5)?????? need help

Answers

Answer:

k=-6

Step-by-step explanation:

Answer:

[tex]\Large \boxed{k=-6}}[/tex]

Step-by-step explanation:

35 = -5(2k+5)

Expand brackets.

35 = -10k - 25

Add 25 on both sides.

60 = -10k

Divide both sides by -10.

-6 = k

Solve for x show work plz

Answers

Answer:

20

Step-by-step explanation:

We know that ΔBAC and ΔBED are similar becuase of the AA Postulate (they both have a right angle and they both share ∠B). Since AC is 15/3 = 5 times longer than ED, the scale factor is 5 which means AB is 5 times longer than EB. This makes EB = 25 / 5 = 5. Since AB = AE + EB and we know that AB = 25, AE = x and EB = 5, 25 = x + 5 which means x = 20.

Did u figure it out?

For triangle DEF, angle D = 42 degrees, line e = 30 meters and line d = 25 meters. Determine the number of possible triangles that can be constructed. Show work.

Answers

Answer:

  2 triangles

Step-by-step explanation:

The given angle is opposite the shorter of the given sides, so the number of triangles is 2. (30/25·sin(42°) ≈ 0.8 < 1)

_____

Additional comment

For the case where the shorter given side is opposite the given angle, there is the possibility that the triangle could be a right triangle (1 solution) or that there may be no solutions. You can tell the difference by computing ...

  (long side)/(short side) × sin(given angle)

If this result is exactly 1, the triangle is a right triangle. If it is greater than 1, the triangle cannot exist (no solutions). Since the sines of most angles are irrational, it is unlikely you will see this result be exactly 1 (except for a 30°-60°-90° right triangle).

These observations are a consequence of the Law of Sines, which tells you ...

  sin(A) = (a/b)sin(B)

For real angles, sin(A) ≤ 1.

2 triangles hope this helps

What’s x ?? Help plzzz

Answers

Answer:

x = 69°

Step-by-step explanation:

1). JFGHI is a pentagon, every angle in a Pentagon is equal to 108°.

2). angle JIH + angle HIE = 180°; angle JIH = 108, so angle HIE = 72°.

3).  angle JDI is equal to 39° and angle IEH; so angle IEH = 39°

4). Now we know 2 out of the three angles in the triangle IHE, so we can find x!

5). x + angle HIE + angle IEH = 180°

    x + 72° + 39° = 180°

    x = 69°

Find the distance between (-5,-6) and (-3,-8 WILL GIVEBRANLIEST TO FIRST PERSON WHO AWNSES WITH EXPLANATION

Answers

Answer:

d = √8

d ≈ 2.82843

Step-by-step explanation:

Distance Formula: [tex]d = \sqrt{(x_2-x_1)^2+(y_2-y_1)^2}[/tex]

Simply plug in our coordinates into the distance formula:

[tex]d = \sqrt{(-3 + 5)^2+(-8 + 6)^2}[/tex]

[tex]d = \sqrt{(2)^2+(-2)^2}[/tex]

[tex]d = \sqrt{4+4}[/tex]

[tex]d = \sqrt{8}[/tex]

To find the decimal, simply evaluate the square root:

√8 = 2.82843

Answer:

[tex] \boxed{2 \sqrt{2} \: \: units}[/tex]

Step-by-step explanation:

Let the points be A and B

A ( - 5 , - 6 ) ⇒ ( x₁ , y₁ )

B ( -3 , - 8 )⇒( x₂ , y₂ )

Now, let's find the distance between these two points:

AB = [tex] \mathsf{ \sqrt{ {(x2 - x1)}^{2} + {(y2 - y1)}^{2} } }[/tex]

Plug the values

⇒[tex] \mathsf{ \sqrt{( - 3 - ( - 5) )^{2} + {( - 8 - ( - 6))}^{2} } }[/tex]

When there is a ( - ) in front of an expression in parentheses, change the sign of each term in the expression

⇒[tex] \mathsf{ \sqrt{ {( - 3 + 5)}^{2} + {( - 8 + 6)}^{2} } }[/tex]

Calculate

⇒[tex] \mathsf{ \sqrt{ {(2)}^{2} + {( - 2)}^{2} } }[/tex]

Evaluate the power

⇒[tex] \mathsf{ \sqrt{4 + 4} }[/tex]

Add the numbers

⇒[tex] \mathsf{\sqrt{8} }[/tex]

Simplify the radical expression

⇒[tex] \mathsf{ \sqrt{2 \times 2 \times 2}} [/tex]

⇒[tex] \mathsf{2 \sqrt{2} }[/tex] units

Hope I helped!

Best regards!

PLEASE HELP! 10 POINTS Which line would be the line of best fit for the scatter plot?

Answers

Answer:

The first one

Step-by-step explanation:

This line is the best fit for the scatter plot because it touches a lot more points than the second

Answer:

The first line

Step-by-step explanation:

Hey there!

Well the first lie is a positive slope just like the dots whereas,

the second line is a negative slope.

Therefore, the first line is the line of best fit.

Hope this helps :)

Tamara walked 3/4 mile in 1/2 hour. Which of the following represents the unit rate that Tamara walked? A. 1/2 mi/h B. 2/3 mi/h C. 3/4 mi/h D. 1 1/2 mi/h Include ALL work please!

Answers

Answer:

1 1/2 miles / hour

Step-by-step explanation:

We want distance / time

3/4 miles

---------------

1/2 hour

3/4 ÷ 1/2

Copy dot flip

3/4 * 2/1

3/2

1 1/2 miles / hour

Answer:

D

Step-by-step explanation:

Multiple (3/4)  by 2 to find the information for one hour)

6/4 in one hour

3/2 or 1 + 1/2 mi/h

D is the answer

f(x) = [tex]\sqrt{x+7} -\sqrt{x^2+2x-15}[/tex] find the domain

Answers

Answer:

x >= -7  ................(1a)

x >= 3   ...............(2a1)

Step-by-step explanation:

f(x) =  [tex]\sqrt{x+7}-\sqrt{x^2+2x-15}[/tex]  .............(0)

find the domain.

To find the (real) domain, we need to ensure that each term remains a real number.

which means the following conditions must be met

x+7 >= 0  .....................(1)

AND

x^2+2x-15 >= 0 ..........(2)

To satisfy (1),  x >= -7  .....................(1a) by transposition of (1)

To satisfy (2), we need first to find the roots of (2)

factor (2)

(x+5)(x-3) >= 0

This implis

(x+5) >= 0 AND (x-3) >= 0.....................(2a)

OR

(x+5) <= 0 AND (x-3) <= 0 ...................(2b)

(2a) is satisfied with x >= 3   ...............(2a1)

(2b) is satisfied with x <= -5 ................(2b1)

Combine the conditions (1a), (2a1), and (2b1),

x >= -7  ................(1a)

AND

(

x >= 3   ...............(2a1)

OR

x <= -5 ................(2b1)

)

which expands to

(1a) and (2a1)   OR  (1a) and (2b1)

( x >= -7 and x >= 3 )  OR ( x >= -7 and x <= -5 )

Simplifying, we have

x >= 3  OR ( -7 <= x <= -5 )

Referring to attached figure, the domain is indicated in dark (purple), the red-brown and white regions satisfiy only one of the two conditions.

f(x) = 
find the domain

URGENT!!!! PLEASE HELP NOW!!! WHO EVER GIVES THE CORRRECT ANSWER WILL GET BRAINLIEST!!!

Items Sold at a Clothing Store
The bar graph shows the number of each item sold at a clothing store. Which
statement about the graph is true?​

Answers

Answer:

The correct ans is..... ( which i believe )

3rd option

Hope this helps...

Pls mark my ans as brainliest

If u mark my ans as brainliest u will get 3 extra points

Answer:

3rd option

Step-by-step explanation:

because 2/5 of 40 is equal to 16, and thats the equivelent of the pants sold.

Name five fractions whose values are between 3/8 and 7/12

Answers

Answer:

convert them to decimasls

Step-by-step explanation:

convert thhem to decimals to make it easier

Answer:

1/2 2/4 4/8 6/12 9/18

Step-by-step explanation:

*PLEASE ANSWER* Compare the volume of these two shapes,given their radii and heights are the same .

Answers

Answer:

The correct option is;

Left object volume = right object volume

Step-by-step explanation:

The shapes given in the question are two circular cones that have equal base radius and equal height

The formula for the volume, V of a circular cone = 1/3 × Base area × Height

The base area of the two shapes are for the left A = π·r², for the right A = π·r²

The heights are the same, therefore, the volume are;

For the left

[tex]V_{left}[/tex] = 1/3×π·r²×h

For the right

[tex]V_{right}[/tex] = 1/3×π·r²×h

Which shows that

1/3×π·r²×h  = 1/3×π·r²×h and [tex]V_{left}[/tex]  = [tex]V_{right}[/tex], therefore, the volumes are equal and the correct option is left object volume = right object volume.

left object volume = right object volume

help me plz can i have help

Answers

Answer:

[tex]10^{-4}[/tex]

Step-by-step explanation:

6x10^-4 because the decimal is in the thousands place

What is the total surface area of the square pyramid below? A square pyramid. The square base has side lengths of 10 centimeters. The triangular sides have a height of 14 centimeters. 100 cm2 200 cm2 280 cm2 380 cm2

Answers

Answer:

380 cm^2

Step-by-step explanation:

10*10=100(base square)

Each triangle= (14*10) /2=70

4 sides, so therefore: 70*4=280

280+100=380

Since the square base has side 10 cm and triangular side has height 14 cm, the total surface area of square pyramid is 380 cm²

To answer the question, we need to find the total surface area of the square pyramid

Total surface area of a square pyramid.

Since a square pyramid has a square base and four triangular sides, its surface area, A = area of square base + 4 × Area of triangular side.

Area of square base

Area of square with side, L is A = L²

Area of triangular base

Area of triangle with height, h and base, L which is the length of the square base is A' = 1/2Lh

So, total surface area of square pyramid is A" = L² + 4 × 1/2Lh

= L² + 2Lh

Given that the length of the square base is 10 cm and the height of the triangular side is 14 cm.

We have that

L = 10 cm and h = 14 cm

So, substituting the values of the variables intot he equation, we have

A" = L² + 2Lh

A" = (10 cm)² + 2 × 10 cm × 14 cm

A" = 100 cm² + 280 cm²

A" = 380 cm²

So, the total surface area of square pyramid is 380 cm²

Learn more about total surface area of square pyramid here:

https://brainly.com/question/22744289

#SPJ2

please help me.... The question no.b and would like to request you all just give me correct answer. ​

Answers

Answer:  see proof below

Step-by-step explanation:

You will need the following identities to prove this:

[tex]\tan\ (\alpha-\beta)=\dfrac{\tan \alpha-\tan \beta}{1+\tan \alpha\cdot \tan \beta}[/tex]

[tex]\cos\ 2\alpha=\cos^2 \alpha-\sin^2\alpha[/tex]

LHS → RHS

[tex]\dfrac{2\tan\ (45^o-A)}{1+\tan^2\ (45^o-A)}\\\\\\=\dfrac{2\bigg(\dfrac{\tan\ 45^o-\tan\ A}{1+\tan\ 45^o\cdot \tan\ A}\bigg)}{1+\bigg(\dfrac{\tan\ 45^o-\tan\ A}{1+\tan\ 45^o\cdot \tan\ A}\bigg)^2}\\\\\\=\dfrac{2\bigg(\dfrac{1-\tan\ A}{1+\tan\ A}\bigg)}{1+\bigg(\dfrac{1-\tan\ A}{1+\tan\ A}\bigg)^2}\\\\\\=\dfrac{2\bigg(\dfrac{1-\tan A}{1+\tan A}\bigg)}{1+\bigg(\dfrac{1-2\tan\A+\tan^2 A}{1+2\tan A+\tan^2A}\bigg)}\\[/tex]

[tex]=\dfrac{2\bigg(\dfrac{1-\tan A}{1+\tan A}\bigg)}{\dfrac{(1+2\tan A+\tan^2A)+(1-2\tan A+\tan^2 A)}{1+2\tan A+\tan^2A}}\\\\\\=\dfrac{2\bigg(\dfrac{1-\tan A}{1+\tan A}\bigg)}{\dfrac{2+2\tan^2A}{1+2\tan A+\tan^2A}}\\\\\\=\dfrac{2\bigg(\dfrac{1-\tan A}{1+\tan A}\bigg)}{2\bigg(\dfrac{1+\tan^2A}{(1+\tan A)^2}\bigg)}\\\\\\=\dfrac{\bigg(\dfrac{1-\tan A}{1+\tan A}\bigg)}{\bigg(\dfrac{1+\tan^2A}{(1+\tan A)^2}\bigg)}[/tex]

[tex]=\dfrac{1-\tan A}{1+\tan A}}\times \dfrac{(1+\tan A)^2}{1+\tan^2A}\\\\\\=\dfrac{1-\tan^2 A}{1+\tan^2 A}\\\\\\=\dfrac{1-\dfrac{\sin^2 A}{\cos^2 A}}{1+\dfrac{\sin^2 A}{\cos^2 A}}\\\\\\=\dfrac{\bigg(\dfrac{\cos^2 A-\sin^2 A}{\cos^2 A}\bigg)}{\bigg(\dfrac{\cos^2 A+\sin^2 A}{\cos^2 A}\bigg)}\\\\\\=\dfrac{\cos^2 A-\sin^2 A}{\cos^2 A+\sin^2 A}\\\\\\=\dfrac{\cos^2 A-\sin^2 A}{1}\\\\\\=\cos^2 A-\sin^2 A\\\\\\=\cos 2A[/tex]

cos 2A = cos 2A   [tex]\checkmark[/tex]

Cos2A = Cos2A
I hope this helps!

What is 5 divided by 3,678

Answers

Answer:

Simple division..

divide 5 by 3,678

you'll get answer

0.00135943447

●✴︎✴︎✴︎✴︎✴︎✴︎✴︎✴︎❀✴︎✴︎✴︎✴︎✴︎✴︎✴︎✴︎✴︎●

      Hi my lil bunny!

❧⎯⎯⎯⎯⎯⎯⎯⎯⎯⎯⎯⎯⎯⎯⎯⎯⎯⎯⎯⎯⎯⎯⎯⎯⎯⎯⎯⎯⎯⎯⎯⎯⎯⎯⎯⎯⎯⎯☙

[tex]5\ \div \ 3678[/tex]

[tex]= \frac{5}{3678}[/tex] (Decimal: 0.001359)

●✴︎✴︎✴︎✴︎✴︎✴︎✴︎✴︎❀✴︎✴︎✴︎✴︎✴︎✴︎✴︎✴︎✴︎●

If this helped you, could you maybe give brainliest..?

❀*May*❀

A building casts a 33-m shadow when the sun is at an angle of 27° the vertical. How tall is the building to the
nearest meter? How far is it from the top of the building to the tip of the shadow?

Answers

Answer:

1. EF = 65m

2. DF = 73m

Step-by-step explanation:

1. EF = height of the building = h = 33 / tan 27 = 65m

2. DF = sqrt (65² + 33²) = 73m

The building is 64.76 meters long and 73 meters far from the top of the building to the tip of the shadow.

From the triangle DEF, we find the value of EF by using tan function.

tan function is a ratio of opposite side and adjacent side.

tan(27)= 33/FE

0.5095 = 33/FE

Apply cross multiplication:

FE=33/0.5095

FE=64.76

Now DF is the hypotenuse, we find it by using pythagoras theorem.

DF²=DE²+EF²

DF²=33²+64.76²

DF²=1089+4193.85

DF²=5282.85

Take square root on both sides:

DF=72.68

In a triangle the the sum of three angles is 180 degrees.

∠D + 27 +90 =180

∠D + 117 =180

Subtract 117 from both sides:

∠D =63 degrees.

Hence, the building is 64.76 meters long and 73 meters far from the top of the building to the tip of the shadow.

To learn more on trigonometry click:

https://brainly.com/question/25122835

#SPJ4

solve for m. √m-7 = n+3 It is worth like 40 points

Answers

Answer:

sqrt of (m-7) = n+3

m = (n+3)^2 + 7 or m= n^2 + 6n + 16

sqrt of (m)-7 = n+3

m = (n+10)^2 or m =n^2 + 20n + 100

Step-by-step explanation:

(2) move the constants to the other side, and square

or  (1) square and move constants

then you can solve for m

Answer:

[tex]n^2+6n+16[/tex]

Step-by-step explanation:

I'm going to assume you meant [tex]\sqrt{m-7} = n+3[/tex], not  [tex]\sqrt{m} - 7 = n+3[/tex].

[tex](\sqrt{m-7}) ^2 = (n+3)^2\\\\(\sqrt{m-7}^2) = (n+3)^2\\\\(m-7)^{\frac{2}{2} } = (n+3)^2\\\\m - 7 = (n+3)^2\\\\m - 7 = n^2 + 2n\cdot3 + 3^2\\\\m - 7 = n^2 + 6n + 9\\\\m - 7 + 7 = n^2 + 6n + 9 + 7\\\\m = n^2 + 6n + 16[/tex]

Hope this helped!

-5+2(10b-2)=31 need help thanks!

Answers

Answer:

[tex]\Large \boxed{b=2}[/tex]

Step-by-step explanation:

-5+2(10b-2)=31

Expand brackets.

-5+20b-4=31

-9+20b=31

Add 9 on both sides.

20b=40

Divide both sides by 20

b=2

Answer:

b = 2

Step-by-step explanation:

-5+2(10b-2)=31

Multiply the terms in the bracket

That's

- 5 + 20b - 4 = 31

Group like terms

Send the constants to the right side of the equation

That's

20b = 31 + 4 + 5

Simplify

20b = 40

Divide both sides by 20

We have the final answer as

b = 2

Hope this helps you

Find the measure of the remote exterior angle. m∠x=(4n−18)°m∠y=(n+9)°m∠z=(151−5n)° A. 71 B. 16 C. 100 D. 46

Answers

Answer: A.71

Step-by-step explanation: 1.set up the equation

2. 4n-18+n+9=151-5n

3.combine like terms ->  5n-9=151-5n

4.solve for n

5.10n=160

n=16

now you have to plug 16 in for n in order to get the remote exterior angle.

151-5(16)

151-80

71°

The sum of any two interior angle is equal to the third exterior angle. The value of n will be 16°. Then the correct option is B.

What is the triangle?

A triangle is a three-sided polygon with three angles. The angles of the triangle add up to 180 degrees.

The triangle having the interior angle x and y, and z is the exterior angle of triangle.

m∠x = (4n − 18)°, m∠y = (n + 9)°, and m∠z = (151 − 5n)°

Then the value of n will be

We know that the sum of any two interior angle is equal to the third exterior angle. Then we have

∠x + ∠y = ∠z

4n - 18 + n + 9 = 151 - 5n

10n = 160

n = 16°

Then the correct option is B.

More about the triangle link is given below.

https://brainly.com/question/25813512

#SPJ2

Find the number set which satisfies each of the problems. If 7 is subtracted from the absolute value of the sum of a number and 6, the result is 3.

Answers

Answer:

x=4      or    x= - 16

Step-by-step explanation:

|x+6|

Now subtract 7 which equals to 3.

|x+6|-7=3

|x+6|=10

Now remove the mode by adding plus minus sign in the front of 10.

x+6=±10

x+6=10 or x+6=-10

x=4      or x=-16

It is 4 or 16 you can put either one


Find the sine and cosine of each angle as a fraction and as a decimal. Round to the nearest hundredth.

Answers

Answer:

1st triangle:

sin(C): 0.64, [tex]\frac{16}{25}[/tex]

cos(C): 0.8, [tex]\frac{4}{5}[/tex]

Second triangle:

sin(C): 0.75, [tex]\frac{\sqrt{5} }{3}[/tex]

cos(C): 0.67, [tex]\frac{2}{3}[/tex]

Step-by-step explanation:

Using SOH CAH TOA, we know that to find the sin of an angle, it's Opposite/Hypotenuse.

To find the cos of an angle, it's adjacent/hypotenuse.

In the 1st triangle:

The adjacent to C is 16, the hypotenuse is 25.

[tex]\frac{16}{25} = 0.64[/tex] is the sin of C.

The adjacent to C is 20, and the hypotenuse is 25.

[tex]\frac{20}{25} = 0.8[/tex] is the cos of C.

In the second triangle:

The opposite to C is [tex]\sqrt{5}[/tex] and the hypotenuse is 2.

[tex]\frac{\sqrt{5} }{2} \approx0.75[/tex] is the sin of C.

The adjacent to C is 2 and the hypotenuse is 3.

[tex]\frac{2}{3} \approx 0.67[/tex] is the cos of C.

Hope this helped!

0.64 because it makes the most sense to me well because I looked it up too

Simplify (−3c3w5)3. −9c6w8 −9c9w15 −27c6w8 −27c9w15

Answers

Answer:

-6723cw

Step by Step:

(-3c * 3w * 5) * 3 - 9c * 6w* 8 - 9c * 9w * 15 - 27c * 6w * 8 - 27c * 9w * 15

The answer to you question is -6723cw
Other Questions
5.(a) In a class of 37 students, the number of students who like marshal arts only isdouble than the number of students who like athletics only. If 3 students like bothand 4 like none of the games, find out how many students like:i. Marshal artsii. Athletics the school bought a sandbox that measures 50 meters long 25 meters wide and 5 meters tall how many cubic meters if sand would you need to buy if each cubic meter of sand cost $1.50 how much money would it cost to fill the sandbox A 384 Hz tuning fork produces standing waves with a wavelength of 0.90 m inside a resonance tube. The speed of sound at experimental conditions is A triangle has one side that lies along the line y=1/4x and another that lies along the line y=-1/4x. Which of the following points could be a vertex of the triangle? An online polling site posed this question: "How much stock do you put in long-range weather forecasts?" Among its Web site users, 38, 528 chose to respond Complete parts (a) through (c) below. a. Among the responses received, 3% answered with "a lot". What is the actual number of responses consisting of "a lot"? b. Among the responses received, 18, 566 consisted of "very little or none". What percentage of responses consisted of "very little or none"? c. Because the sample size of 38, 528 is so large, can we conclude that about 3% of the general population puts "a lot" of stock in long-range weather forecasts? Why or why not? A. No, because the sample is a voluntary response sample, so the sample is not likely to be representative of the population. B. Yes, because the sample is so large, the margin of error is negligible. C. No, because even though the sample size is so large, there is still a margin of error. D. Yes, because the sample size is large enough so that the sample is representative of the population. Suppose the price level and value of the U.S. Dollar in year 1 are 1 and $1, respectively. Instructions: Round your answers to 2 decimal places. a. If the price level rises to 1.35 in year 2, what is the new value of the dollar? Compute the range and interquartile range for the data collected for boys and girls. Describe their differences in detail using specific terms of spread. (4 points) Candidates for depth interviews could include a. current customers. b. members of the target market. c. executives and managers of the company. d. All of these are candidates for depth interviews. Reduce the following fraction to lowest terms: 8/14 A 25.00 mL sample of unknown concentration of HNO3 solution requires 22.62 mL of 0.02000 M NaOH to reach the equivalence point. What is the concentration of the unknown HNO3 solution Plz help me this is probably easy but im just not seeing it plz help me ASAP Vasudevan Inc. recently reported operating income of $2.90 million, depreciation of $1.20 million, and had a tax rate of 40%. The firm's expenditures on fixed assets and net operating working capital totaled $0.6 million. How much was its free cash flow, in millions? Cite algumas medidas tomadas por Getlio Vargas no incio de seu mandato e explique de que maneira elas culminaram na Revoluo Constitucionalista de 1932 What are an example of push factors and pull factors? Which of the c-values satisfy the following inequality? 2>c/3 Identify the location of the Harappan civilization on this map of the Indian subcontinent. How did the process of selective incorporation influence the U.S. governmentin the 20th century?O A. It gave states more power to operate independently from thefederal government.O B. It allowed the government to place more restrictions on individualcivil liberties.C. It ensured that state governments had to respect most Bill ofRights protections.D. It made it easier for citizens to demand amendments to the U.S.Constitution. What would be the pH of an HNO3 solution if the [H+] = 3.26 x 10-6 moles/liter? 5.49 4.67 6.32 its either A or B but idk, ill give brainliest :) Lilliput is a country that has closed borders and does not import or export any goods or services; hence, they do not worry about trade with other countries.Total spending for the federal government of Lilliput for the last fiscal year was $1.06 billion. The country collected $1.05 billion in taxes during this same fiscal year. Assume government transfers were zero. Based on this information, what is Lilliput's budget balance? In the last fiscal year, Lilliput was running:______.a. a budget surplus.b. a balanced budget.c. a budget deficit.